LSAT and Law School Admissions Forum

Get expert LSAT preparation and law school admissions advice from PowerScore Test Preparation.

User avatar
 Dave Killoran
PowerScore Staff
  • PowerScore Staff
  • Posts: 5852
  • Joined: Mar 25, 2011
|
#27285
Complete Question Explanation
(The complete setup for this game can be found here: lsat/viewtopic.php?t=1171)

The correct answer choice is (D)

The impact of the rule suspension is to create uncertainty over whether X has two or three options. W must still have two options since W and Y have no options in common. If X has two options, then X must have options L and P so as to have exactly two options in common with W; if X has three options, then X must of course have options L, P, and S. All other relationships and options remain identical:
Oct 01_M12_game#2_L9_explanations_game#1_#12_diagram_1.png
Because X and Z must have at least one option in common, answer choice (D) cannot be true and is correct.


Note the heavy presence of False To True questions and Except questions in this game. This likely occurs because the game setup contains so much information, leading to a situation where the test makers want to create greater difficulty in the questions (a result usually caused by the False To True wording and the use of Except).
You do not have the required permissions to view the files attached to this post.
 lsnewbie
  • Posts: 12
  • Joined: Aug 31, 2018
|
#60509
Hello PS,
When I read the stimulus for this question, I thought the new condition requiring X and W to have exactly 2 options in common meant that X now only had 2 options (L&P), instead of 3 (L,P&S). With this understanding, I selected C, although I see D is an option too (and actually the correct answer). I didn't realize X could still have 3 options ... I guess the test makers were trying to throw a curveball there. :hmm:
Administrator wrote:Complete Question Explanation

If X has two options, then X must have options L and P so as to have exactly two options in common with X; if X has three options, then X must of course have options L, P, and S. All other relationships and options remain identical:
Also, in the above explanation, did you mean if X has two options, then X must have options L and P so as to have exactly two options in common with W, not X? That may have been a typo. Just thought I would point it out.

Get the most out of your LSAT Prep Plus subscription.

Analyze and track your performance with our Testing and Analytics Package.